Difference between revisions of "2021 AMC 10B Problems/Problem 1"

Line 1: Line 1:
This article was made early. Please ignore it until 2021 AMC 10B Problems are released.
+
How many integer values of <math>x</math> satisfy <math>|x|<3\pi</math>?
 +
 
 +
<math>\textbf{(A)} ~9 \qquad\textbf{(B)} ~10 \qquad\textbf{(C)} ~18 \qquad\textbf{(D)} ~19 \qquad\textbf{(E)} ~20</math>

Revision as of 17:07, 11 February 2021

How many integer values of $x$ satisfy $|x|<3\pi$?

$\textbf{(A)} ~9 \qquad\textbf{(B)} ~10 \qquad\textbf{(C)} ~18 \qquad\textbf{(D)} ~19 \qquad\textbf{(E)} ~20$